Difference between revisions of "2004 AMC 12A Problems/Problem 5"
(New page: ==Problem== The graph of the line <math>y=mx+b</math> is shown. Which of the following is true? Image:2004 AMC 12A Problem 5.png <math>\mathrm {(A)} mb<-1 \qquad \mathrm {(B)} -1<mb<...) |
(→Solution) |
||
Line 7: | Line 7: | ||
==Solution== | ==Solution== | ||
− | It looks like it has a slope of <math>-\dfrac{1}{2}</math> and is shifted <math>\dfrac{4}{5}</math> up. <math>\dfrac{4}{5}\cdot \dfrac{-1}{2}=\dfrac{-4}{10} \Rightarrow \mathrm {(B)}</math> | + | It looks like it has a slope of <math>-\dfrac{1}{2}</math> and is shifted <math>\dfrac{4}{5}</math> up. |
+ | |||
+ | <math>\dfrac{4}{5}\cdot \dfrac{-1}{2}=\dfrac{-4}{10} \Rightarrow \mathrm {(B)}</math> | ||
==See also== | ==See also== |